LSAT and Law School Admissions Forum

Get expert LSAT preparation and law school admissions advice from PowerScore Test Preparation.

 Administrator
PowerScore Staff
  • PowerScore Staff
  • Posts: 8919
  • Joined: Feb 02, 2011
|
#66030
Complete Question Explanation

The correct answer choice is (B).

Answer choice (A):

Answer choice (B): This is the correct answer choice.

Answer choice (C):

Answer choice (D):

Answer choice (E):


This explanation is still in progress. Please post any questions below!
 val12
  • Posts: 3
  • Joined: Nov 16, 2019
|
#73147
I first looked at M because of its conditional relationship with L. My thinking process was since M has had a pretty big impact on the game thus far, I'll just try it out. Is there a better strategic approach to Justify questions like this one?
 cargostud
  • Posts: 17
  • Joined: Dec 23, 2019
|
#73157
I can't think of another way to figure it out other than the logic you presented. As I analyze it -

Friday - If M works on Friday that will force L to work on Friday. So Friday fills up with MLN (Since we already know N works on Friday).

Thursday - Since we already have L working on Friday and Saturday, L can't work on Thursday and neither can M, so that leaves NPQ for Thursday. (L working on Saturday is a setup inference)

Saturday - No P; No N (N is already on Thur and Fri). L is inferred from the setup. That leaves M and Q. So we have LMQ

I think my new motto will be: Yay! I figured it out, but the timer ended before I could select the answer.
 cargostud
  • Posts: 17
  • Joined: Dec 23, 2019
|
#73158
Here is a little diagram I made for this game. Along with the rules, this seemed to help me.

L M N P Q (Any of these can work on Thursday)
\ \ | / /
Thursday

L M N P Q (N works on Friday)
\ \ | / /
Friday

L M N Q (L works on Saturday. P does not work on Saturday)
\ \ | /
Saturday

M :arrow: L
~L :arrow: ~M
L can work even when M is not
2-2-2-2-1
User avatar
 KelseyWoods
PowerScore Staff
  • PowerScore Staff
  • Posts: 1079
  • Joined: Jun 26, 2013
|
#73316
Hi val12!

That's a great strategy for approaching a question like this. You know that you have a rule that says if you place M, you already know for sure where L goes. So that's an excellent place to start. Plus, you have that in combination with placing M on Friday, where N already is, so that day gets filled. When you have questions where you think you might have to try out diagrams for a few different answer choices, it's always best to strategically start with the answer choices that involve variables that you know are already very restricted.

Great job!

Best,
Kelsey
User avatar
 KelseyWoods
PowerScore Staff
  • PowerScore Staff
  • Posts: 1079
  • Joined: Jun 26, 2013
|
#73317
Good setup, cargostud!
 ofc95
  • Posts: 3
  • Joined: Jul 26, 2020
|
#78287
I got (D) for this one, and I'm not sure why it's wrong. Could you walk me through how to solve this question?

Thank you,
Olivia
User avatar
 KelseyWoods
PowerScore Staff
  • PowerScore Staff
  • Posts: 1079
  • Joined: Jun 26, 2013
|
#78376
Hi Olivia!

For a question like this, you are looking for a condition in an answer choice that will cause everything else to fall into place. You can try out answer choices by diagramming the condition in the answer choice and seeing what else must be true based on your rules. For answer choice (D), if you put P into Friday, what else must be true? It must be true that M cannot be on Friday (since there isn't room for L to be with it). But then you're kind of stuck. It seems like either L or Q could be in Friday. And you have no idea which of the volunteers is only going to be used once (since no volunteer can work all 3 days, the numerical distribution of days to volunteers is 2-2-2-2-1). So placing P on Friday doesn't completely determine the assignment of volunteers. There are still too many possibilities.

But if you place M on Friday, then you know that L must also go on Friday. Now Friday is full. Since P can't go on Saturday, P can only go on Thursday and it is, therefore, the only volunteer that is used just once. All other volunteers must go on 2 days. So Q has to go on both Thursday and Saturday. Since there's only one slot left on Thursday, M can't go there (since there's no room for L to be with it). So instead M has to go on Saturday and L has to be there as well. That just leaves N in the 3rd slot on Thursday. Placing M on Friday has completely determined the assignment of volunteers. There is only one possible arrangement of the volunteers if M is on Friday.

Hope this helps!

Best,
Kelsey

Get the most out of your LSAT Prep Plus subscription.

Analyze and track your performance with our Testing and Analytics Package.